![]() |
![]() |
Problem Posts | ![]() |
![]() |
QUESTION
(Adapted from UVA 10784) The number of diagonals of an n-gon is not less than N. Find a closed-form expression for the minimum possible value of n.
(Adapted from UVA 10784) The number of diagonals of an n-gon is not less than N. Find a closed-form expression for the minimum possible value of n.
Laconic Solution Sketch
A polygon has an integral side number. Use bounds.
Discussion
Don't underestimate the power of making a ballpark estimate. We know that for some n, N \le \frac{n \cdot\left(n-3\right)}{2}. Since n is minimum, n-1 won't cut it: \frac{\left(n-1\right) \cdot\left(n-1-3\right)}{2} < N. Now expand: n^2-5n+4<2N\le n^2-3n Our instincts tell us to complete the square. Add 9/4 to each element: \left(n-\frac{5}{2}\right)^2<2N+\frac{9}{4}\le \left(n-\frac{3}{2}\right)^2 It's noteworthy that adding the same constant completes the square for both sides. This is no accident, as the left hand side is just the right hand side under the substitution n \rightarrow n-1. We finish this one off: n-1<\sqrt{2N+\frac{9}{4}}+\frac{3}{2}\le n and so n=\boxed{\left \lceil \sqrt{2N+\dfrac{9}{4}}+\dfrac{3}{2} \right \rceil}.
Warning: Note how we're "forced" to use the least integer (ceiling) function instead of the greatest integer function, because the non-strict inequality lies between the second and third expressions. Of course, we could use the floor function, but it's cumbersome: n=\boxed{-\left \lfloor -\left( \sqrt{2N+\dfrac{9}{4}}+\dfrac{3}{2} \right) \right \rfloor} Admittedly, both expressions are cumbersome. The question comes up:
BONUS QUESTION 1
If we had the added information that N>0 is indeed the number of diagonals of some polygon, could we make a simpler expression - simple enough to calculate mentally?
If we had the added information that N>0 is indeed the number of diagonals of some polygon, could we make a simpler expression - simple enough to calculate mentally?
Yes. Since we know that N=\frac{n \cdot\left(n-3\right)}{2}>0, we can afford to be lax and choose bounds that we like (bounds that don't involve fractions). First, N>0 hence n>3. We bound 2N between two perfect squares: n^2-4n+4 \le n^3 - 3n < n^2-2n+1 (We can verify that 4-n\le 0 < n+1, and we simply add 2N to each side). Then n=\boxed{\left\lfloor 2N \right \rfloor +2} which is much nicer.
BONUS QUESTION 2
(Mr Jurgeen Vasquez, OEIS) Find a closed-form expression for the n^\mathrm{th} term a_n of the sequence 1,1,2,1,2,3,1,2,3,4\ldots
(Mr Jurgeen Vasquez, OEIS) Find a closed-form expression for the n^\mathrm{th} term a_n of the sequence 1,1,2,1,2,3,1,2,3,4\ldots
The most sensible thing to do is arrange the sequence in tiers: \begin{array}{cccc} 1,\\ 1, & 2,\\ 1, & 2, & 3,\\ 1, & 2, & 3, & 4,\ldots \end{array} Now we want a formula to know which tiera_n term is in. Now, if a_n is in the N^\mathrm{th} tier, we have \frac{N \cdot\left(n-1\right)}{2} < n\le\frac{N \cdot\left(N+1\right)}{2} Using exactly the same method as above we get N=\left\lceil \sqrt{2n+\frac{1}{4}}-\frac{1}{2} \right\rceil We figure out that a_n = n - \frac{N^2-N}{2}.
No comments:
Post a Comment